Which statement best describes its usefulness in deciding whether a given situation is abnormal?

Exam 4 P/S Solutions: Passage 1

1) This is a passage-based question that relies on us going back to the passage to pick out some key information. The author tells us in Paragraph 1, “In lexical decision tasks which ask participants to decide whether a string of letters is a word, Ps respond similarly to emotional and non-emotional words, whereas controls tend to respond more quickly to emotional words.” Quick glance at our answer choices shows we’re seeing average reaction time along the y-axis. We’re comparing the reaction time to emotional vs. non-emotional words for the Ps and controls groups. Based on what we learned from the passage, for the Ps group we expect the average reaction time to be roughly the same for emotional and non-emotional words. For the Controls, we expect the average reaction time to be shorter for emotional words and longer for non-emotional words. 

  1. Answer choice A reverses what we said in the breakdown. We expect the Ps group to show little difference in average reaction time between emotional words and non-emotional words. We expect the average reaction time to be shorter for emotional words and longer for non-emotional words.
  2. Answer choice B shows both groups having quicker reaction times to emotional words. While this is consistent with what we know about the Controls, this is not what we expect from the Ps group.
  3. This answer choice correctly shows a similar reaction time to both sets of words for the Ps group. However, it incorrectly lists average reaction time as longer for emotional words for the Controls. We expect the opposite to be true for the Controls
  4. This answer choice matches our breakdown exactly. For the Ps group we expect the average reaction time to be roughly the same for emotional and non-emotional words. For the Controls, we expect the average reaction time to be shorter for emotional words and longer for non-emotional words. Answer choice D is going to be our best answer choice.

2) This question is tangentially related to the passage. In Paragraph 2, the author mentions that Ps were much less likely than controls to inhibit responses and to acquire avoidance learning. We want an experimental procedure that helps look into this. How can researchers test how Ps perform versus controls in various types of operant conditioning? We’ll have to define these terms. Ultimately our answer is going to involve comparing the Ps and controls in some way. We’re focused on operant conditioning. 

Operant conditioning is a theory of learning that focuses on changes in an individual’s observable behaviors. In operant conditioning, new or continued behaviors are impacted by new or continued consequences.

Which statement best describes its usefulness in deciding whether a given situation is abnormal?

Avoidance learning is the process by which an individual learns a behavior or response to avoid a stressful or unpleasant situation. The behavior is to avoid, or to remove oneself from, the situation. The reinforcement for the behavior is to not experience the negative punishment, but rather experience the absence of punishment.

  1. on a word-learning task in which they receive a monetary reward every time they provide the correct response. This answer choice describes operant conditioning. The participants are asked to provide a correct response and reinforcement follows. 
  2. on a task that requires watching a model perform a series of physical actions and repeating them in the correct order. This answer choice does not describe operant conditioning, but rather observational learning. Observational learning is learning that occurs as a function of seeing, retaining, and, in the case of imitation learning, replicating novel behavior executed by other people. Answer choice A remains superior.
  3. in a procedure in which the participants are presented with a light followed by a drop of lemon juice in their mouths which elicits salivation. This answer choice describes classical conditioning. Classical conditioning is learning in which the stimulus or experience occurs before the behavior and then gets paired or associated with the behavior.
  4. in a procedure in which the participants are presented with a word list and tested on their recall two days later. This answer choice is asking participants to use their memory to recall words. Recall is information retrieved from memories. We can stick to answer choice A as our best answer choice. 

3) Just like the other questions we’ve gone through in this set, this is a passage-based question that relies on us going back to the passage to pick out key information. The author highlights these key differences in Paragraph 1:

The controls had an involuntary increase in moisture (sweating) which means increased skin conductivity. This is an indication of psychological and physiological arousal. This involuntary sweating is stimulated by the sympathetic nervous system (SNS). The SNS is perhaps best known for mediating the neuronal and hormonal stress response commonly known as the fight-or-flight response. The SNS controls the body’s automatic response to danger, increasing the heart rate, dilating the blood vessels, slowing digestion, and moving blood flow to the heart, muscles, and brain. This ties into the startle response when presented with fear-inducing pictures. Participants in the Ps group are less likely to exhibit that startle response compared to controls. 

  1. sympathetic nervous system arousal. This answer choice is consistent with our breakdown of the question. The SNS controls the body’s automatic response to danger, increasing the heart rate, dilating the blood vessels, slowing digestion, and moving blood flow to the heart, muscles, and brain. We said this explains the differences between Ps and controls when anticipating aversive events and viewing fear-inducing pictures.
  2. somatic nervous system arousal. Somatic nervous system is the part of the peripheral nervous system that transmits signals from the central nervous system to skeletal muscle and from receptors of external stimuli to the central nervous system, thereby mediating sight, hearing, and touch. This is not the difference in the two groups.
  3. parietal cortex activation. Parietal cortex controls somatic or voluntary sensory functions. This is an inferior answer choice to option A. 
  4. occipital cortex activation. Occipital cortex controls vision. Answer choice A remains our best answer choice.

4) This question is similar to our previous questions in that we’ll revisit the passage for key information. We’re asked about the numbers presented to the participants in the avoidance study:

Five numbers were associated with a reward, while five were associated with electric shock. Within the context of avoidance learning, what do these numbers represent? 

  1. reinforcers. Reinforcement and punishment are principles of operant conditioning that increase or decrease the likelihood of a behavior. Reinforcement means you are increasing a behavior: it is any consequence or outcome that increases the likelihood of a particular behavioral response. This sounds more like the monetary incentive. This is not what the numbers function as in this case. 
  2. punishers. Reinforcement and punishment are principles of operant conditioning that increase or decrease the likelihood of a behavior. Punishment means you are decreasing a behavior: it is any consequence or outcome that decreases the likelihood of a behavioral response. This sounds like the electric shock. This is not what the numbers function as in this case.
  3. discriminating stimuli. When an organism learns to respond differently to various stimuli that are similar, it is called stimulus discrimination. The numbers represent discriminating stimuli. The participants were presented with numbers, but they had to respond correctly in order to receive the monetary incentive. Participants had to discriminate between stimuli so this is a strong answer choice. 
  4. unconditioned stimuli. An unconditioned stimulus is a stimulus that elicits a reflexive response. The numbers do not represent an unconditioned stimulus that elicits a reflexive response. Instead the numbers function as discriminating stimuli. Answer choice C is our best option. 

Exam 4 P/S Solutions: Passage 2

5) This is a passage-based question that relies on us going back to the passage to pick out key information. The author actually makes our lives easier in this question because we’re actually told where in the passage we’ll be focusing our attention. 

The author explains that individuals’ beliefs regarding whether their behaviors will lead to positive and negative outcomes also motivate behavior. Social-cognitive theories of personality emphasize the role of cognitive processes, such as thinking and judging, in the development of personality. We want to know if this is consistent with a traditional behaviorist approach. 

A behaviorist will regard an individuals’ actions as ultimately being responses to external stimuli, driven by outcomes. A traditional behaviorist perspective sees outcomes, and not cognition, as the drive behind an individual’s choices and behaviors. Social learning theory believes that personality and behavior are determined by an individual’s cognition about the world around them. Ultimately, the behaviorist perspective is concerned with how environmental factors affect observable behavior. Beliefs fall under mental processes which are inconsistent with a traditional behaviorist approach. We’re looking for an answer that says the role of beliefs is inconsistent with a traditional behaviorist approach. 

  1. Yes, because the traditional behaviorist approach also emphasizes changes in behavior due to desirable and undesirable outcomes. We mentioned in the breakdown of the question, we’re looking for an answer that says the role of beliefs is inconsistent with a traditional behaviorist approach. Beliefs fall under mental processes which are inconsistent with a traditional behaviorist approach. The traditional behaviorist approach suggests outcomes determine behavior. 
  2. Yes, because the traditional behaviorist approach holds that repeated exposure to certain outcomes shapes beliefs regarding these outcomes. Similar to answer choice A, we mentioned in the breakdown of the question, we’re looking for an answer that says the role of beliefs is inconsistent with a traditional behaviorist approach. Beliefs fall under mental processes which are inconsistent with a traditional behaviorist approach. Both answer choices A and B are not strong answers. 
  3. No, because the traditional behaviorist approach holds that mental states are involved only in classical conditioning, not operant conditioning. This answer choice starts as a better choice than answers A and B. Classical conditioning occurs when an appropriate response to a stimulus becomes conditioned to respond to another associated stimulus. Operant conditioning is a theory of learning that focuses on changes in an individual’s observable behaviors. SCT talks about mental states motivating behaviors which is inconsistent with the traditional behaviorist approach.
  4. No, because the traditional behaviorist approach holds that actual outcomes determine behavior, not cognitions regarding potential outcomes. This answer choice is consistent with our breakdown. SCT emphasizes the role of cognitive processes like thinking and beliefs in the development of personality. Alternatively, the behaviorist perspective is concerned with how environmental factors affect observable behavior. 

6) Our approach to this question is going to be similar to our approach to the previous question. This is a passage-based question that relies on us going back to the passage to pick out key information, but ultimately, we’ll need to rely on knowing our vocabulary to answer this question correctly. We can keep Paragraph 4 in mind as we go through each term. Paragraph 4 is when the author explains self-determination theory (SDT). The author mentions controlled behaviors are due to pressure from social contacts (external motivation). Extrinsically motivated behaviors are performed in order to receive something from others—such as a promotion, praise, candy, money, or attention. Extrinsic motivation is therefore a broader term that can actually include external motivation.

  1. is a broader term that includes external motivation. This answer choice matches our breakdown of the question. We said extrinsically motivated behaviors are performed in order to receive something from others. That can be in addition to appeasing social contacts. That pressure from social contacts falls under the umbrella of extrinsic motivators, but there are additional motivators.
  2. is a narrower term that refers to external reinforcers. This is the opposite of our breakdown. We said extrinsically motivated behaviors are performed in order to receive something from others. That includes external motivation, so extrinsic motivation is a broader term, not a narrower term.
  3. excludes social punishers and reinforcers. Reinforcement and punishment are principles of operant conditioning that increase or decrease the likelihood of a behavior. This also falls under extrinsic motivation.
  4. excludes internal states that direct behavior. This is not something that is different about external motivation versus extrinsic motivation. Internal states are excluded from both. We can stick with answer choice A as our best answer. 

7) At first glance, this is like a hybrid question in that we’ll have to reference some key information from the passage, but ultimately, we’ll use our external information to answer the question. The author tells us in Paragraph 4, “Healthcare professionals can increase autonomous motivation by supporting their patients’ initiatives and offering them different choices for treatment.” We want to find a statement that best applies Roger’s concept of incongruence to this suggestion. Carl Rogers believed that people strive to become self-actualized—the “best version” of themselves or “ideal self”. The gap between the “ideal” self and the real self can cause discomfort, unpleasant feelings, and lead to defensive behaviors – this gap is called incongruence.

  1. By encouraging their patients’ initiative, healthcare professionals motivate their patients to reduce the gap in their need for self-actualization. Incongruence is the gap between the “real” and “ideal” self that can cause discomfort or unpleasant feelings. This does not have to do with the need for self-actualization. 
  2. By giving their patients options, healthcare professionals make it possible for their patients to reduce the gap between their actual behavior and their expected behavior. Reasoning here is going to be similar to answer choice A. Incongruence is the gap between the “real” and “ideal” self that can cause discomfort; incongruence does not have to do with actual behavior versus expected behavior. 
  3. By encouraging the patients’ initiative, healthcare professionals motivate their patients to reduce the gap between their behaviors and their attitudes. This answer choice is similar to answer choices A and B. Incongruence is the gap between the “real” and “ideal” self that can cause discomfort, but does not refer to a gap between behavior and attitude.
  4. By giving their patients options, healthcare professionals offer their patients ways to reduce the gap between their ideal selves and their actual selves. Bingo, this is exactly what we’ve been looking for. Incongruence is the gap between the “real” and “ideal” self that can cause discomfort. Healthcare professionals offering options to their patients allows for patients to reduce incongruence. Answer choice D is our best option.

8) To answer this question, we’ll have to think about where in the passage the author talks about autonomous motivation. The passage talks about self-determination theory and its focus on autonomous motivation in Paragraph 4. We’re told, “Autonomous motivation refers to self-initiating behaviors that are tied to one’s values and goals.” This is different than controlled behaviors which are performed due to pressure from social contacts or internal psychological pressures like guilt or fear. We’re also told autonomous motivation is more likely to lead to long-term adherence to treatment. 

  1. The desire to reach one’s goals creates a drive that individuals are motivated to fulfill. This is consistent with what we read in the passage. Autonomous motivation refers to self-initiating behaviors that are tied to one’s values and goals. The desire to reach these goals creates the drive that individuals are motivated to fulfill. This is a strong answer choice.
  2. Living a life consistent with one’s values is socially reinforced. Autonomous motivation has to do with self-initiating behaviors that are tied to one’s values and goals. Answer choice B is out of scope when talking about autonomous motivation.
  3. Individuals are motivated to perform behaviors that result in appetitive internal states. Autonomous motivation involves self-initiating behaviors that are tied to one’s values and goals. The result of autonomous motivation is not appetitive internal states, it’s actually engaging in behavior that is self-determined and consistent with intrinsic goals.
  4. Individuals are reinforced to perform behaviors that are associated with progress. Autonomous motivation is more focused on initial drives and self-initiating behaviors, not end goals that involve reinforcing individuals to perform behaviors associated with progress. Answer choice A is going to be our best answer choice here.  

Exam 4 P/S Solutions: Questions 9-12

9) What a frustrating situation! We’re told researchers are testing a new cancer drug that they deem effective, but other scientists cannot replicate the findings. We want to explain a possible reason for this unfortunate lack of replicability of the original results. We can go through the four explanations listed as answer choices, define each one, and decide which is the most likely explanation for the lack of replicability of the original results.

  1. Base rate fallacy. The base rate fallacy has to do with incorrectly judging a situation like not considering all the information at a researchers’ disposal. Researchers might erroneously ignore base rates in favor of individuating information. That is not what happens in our question stem  
  2. Hindsight bias is the belief that the event just experienced was predictable, even though it really wasn’t. This is not the reason for the lack of replicability of the original results. 
  3. Observer bias occurs when there is bias on the part of the specific researcher – this could occur because of the researcher’s prior knowledge of the study, for example. This is a strong possibility in this case and the initial results may not have been obtained properly. That bias is what caused the initial conclusion that the drug is effective, despite the results not being replicable. This is a strong answer choice.
  4. Public verifiability is likely a distractor. This is not listed on AAMC’s content outline and not a term commonly used when talking about research results. This answer choice is instead referring to the reason the subsequent scientists are testing the initial drug. We can eliminate this answer choice. Answer choice C remains our best option.

10) No tricky verbiage here; this is a basic standalone question that relies on knowing the function of the retina. The retina is the thin layer of cells at the back of the eyeball where light is converted into neural signals sent to the brain. There are two types of photoreceptors in the retina: rods and cones, named for their general appearance.

Which statement best describes its usefulness in deciding whether a given situation is abnormal?

Rods are strongly photosensitive and are located in the outer edges of the retina. They detect dim light and are used primarily for peripheral and nighttime vision. Cones are weakly photosensitive and are located near the center of the retina. They respond to bright light, and their primary role is in daytime, color vision.

  1. accommodate and focus incoming light rays on the lens. This is not a function of the retina. The cornea and the lens bend light to focus the image on the retina; the iris and pupil regulate the amount of light entering the eye. 
  2. detect light rays and convert them into signals for the brain to process. This is consistent with our breakdown. The retina is the thin layer of cells at the back of the eyeball where light is converted into neural signals sent to the brain. This is our best answer choice.
  3. provide oxygen and nutrients to the vitreous humor. The vitreous humor supports the lens and maintains the shape of the entire eye. Blood vessels provide oxygen and nutrients to the eye. The retina functions to detect light rays and convert into signals for the brain to process. Answer choice B remains superior.
  4. control the amount of light focused on the photoreceptor cells. This is a function of the lens. The main function of the lens is to focus light on the retina and fovea centralis. The lens is dynamic, focusing and re-focusing light as the eye rests on near and far objects in the visual field. We can stick with answer choice B as our best answer choice.

11) This is a standalone question that focuses on a sociology concept: the glass escalator. The glass escalator is a direct contrast to the glass ceiling which suggests there is an unofficial barrier to advancement in a profession. The glass ceiling specifically suggests women are not able to advance in their profession as quickly and regularly as their male counterparts. The glass escalator focuses on the other side and says that men are promoted and advance in their profession much more quickly than their female counterparts, specifically in traditionally female-dominated industries. 

  1. He will be promoted to supervisory roles more quickly than his female counterparts. This is consistent with our breakdown of the question. The glass escalator concept says that men are promoted and advance in their profession much more quickly than their female counterparts, specifically in traditionally female-dominated industries. 
  2. He will lack needed experience for promotion because of gender segregation in tasks. The glass escalator concept does not predict the male employee will lack needed experience for promotion. Women are traditionally subject to task segregation in tasks more than their male counterparts. Answer choice A remains superior.
  3. He will face more extensive role conflict with supervisors than his female counterparts. Role conflict describes the conflict between or among the roles corresponding to two or more statuses held by one individual. This is out of scope when discussion the glass escalator concept. 
  4. He will challenge gender norms in the industry and thus be passed over for promotions. This is the opposite of what the glass escalator concept predicts. The male will actually progress more quickly than his female counterparts in a female-dominated industry. We can stick with answer choice A as our best answer.

12) The test-maker provides us with a setup and some general results about research. We’re told the supplies given to the participants and what the participants did with these supplies. The test-maker tells us only a few participants used a seemingly good solution to the task they were asked to do (use the matchbox as a candle holder). We can go through our four answer choices and define each one and see which option correctly identifies the barrier to problem solving the participants experienced. 

  1. Confirmation bias. Confirmation bias is a barrier to problem-solving. This exists when a person tends to look for information that supports their idea or approach instead of looking at new information that may contradict their approach or ideas. This is not what the participants did in the example.
  2. Functional fixedness. Functional fixedness is a special type of mindset that occurs when the intended purpose of an object hinders a person’s ability to see its potential other uses. It’s the inability to see an object as useful for any other use other than the one for which it was intended. The participants did not think of the matchbox as a possible candle holder because of functional fixedness. This is a strong answer choice. 
  3. Overconfidence effect. Overconfidence effect has more to do with a person’s confidence in their judgment or abilities compared to the objective accuracy of the judgment. Participants can overestimate their judgement. There is no evidence the participants were overconfident. 
  4. Availability heuristic. A heuristic is a rule of thumb, a strategy, or a mental shortcut that generally works for solving a problem (particularly decision-making problems). Availability heuristic is a faulty heuristic in which you decide based on information readily available to you and how quickly you can retrieve this information from memory. This was not a barrier experienced by the participants. We can stick with answer choice B as our best answer. 
  5.  

Exam 4 P/S Solutions: Passage 3

13) This is a passage-based question that relies on us going back to the passage to pick out some key information. The author tells us in Paragraph 1, “Surveys were mailed to 398 employees of an organization who were classified as either managers or assistants. The survey was returned by 42% of the sample (64 managers and 104 assistants).” We want to go through our four answer choices and note a concern about the study given this information and the response rate.

  1. Participants provided four responses to four hypothetical scenarios. This answer choice talks about the validity of the study itself, not necessarily the response rate. The four responses and hypothetical scenarios were given regardless of the response rate.
  2. The participants may differ from the employees who did not respond to the survey. This is a valid concern. A response rate of 42% tells us a lot of employees did not respond to the survey. There is a chance the remaining 58% differ from the employees that did return the survey. This is a superior answer to answer choice A.
  3. The manager and assistant participants may have interpreted the scenarios differently. This is actually what researchers were investigating. These different interpretations are expected based on the different groups. Answer choice B remains the biggest concern about the study as a result of the response rate.
  4. Participants were aware of the status of the target employee. This is a true statement and was done on purpose to ultimately get the experimental results. The response rate does not directly influence this concern. We can eliminate this answer choice. Answer choice B is our best answer.

14) Similar to our last question, this is another passage-based question that relies on us going back to the passage to pick out some key information. The test-maker is simply asking us about the results of the research here. Attribution is the process by which individuals explain the cause of behavior and events. They are classified as either internal or external. Internal attributions include dispositional or personality-based explanations; external attributions emphasize situational factors. 

The author tells us in Paragraph 3, “When participants made in-group judgments and the behavior outcomes were negative, participants mainly attributed the behavior to situational factors. When participants made out-group judgments and the outcomes were positive, participants mainly attributed the behavior to situational factors.” We want to know how these would change during intense group conflict between managers and assistants.

  1. Recognizing the need to reduce conflict, situational attributions would be the most common. Situational attributions would likely reduce conflict, but this is not something we expect to see. In the middle of an intense conflict, the managers and assistants would likely be emotional and unlikely to focus on situational factors. 
  2. The conflict would increase the effects of the in-group and out-group bias. This is going to be the most likely change during times of intense group conflict. In the middle of an intense conflict, the managers and assistants would likely be emotional and increase the effects of the in-group and out-group bias. We would see that if participants make in-group judgements and behavior outcomes are negative, the participants would attribute the behavior to situational factors even more. If the behavior outcomes are positive, the participants would attribute the behavior to dispositional factors. We expect the opposite to be true for out-group judgments. This is a strong answer choice.
  3. In-group bias fighting within the organization would reverse the pattern of attributions found in the study. Conflict and fighting usually leads to biases getting stronger. Participants engaged in conflict are more likely to give in to the pattern of attributions found in the study. Answer choice B remains superior.
  4. The increase in self-reliance during conflict would reduce the in-group and out-group bias effects. If there is intense conflict, we expect the group dynamic strengthens rather than an increase in self-reliance. The managers and assistants will turn to their in-group even more than before during the intense conflict. We can stick with answer choice B as the most likely change. 

15) To answer this question, we can consider the findings in the study which the author breaks down in Paragraph 3. We want to use these findings to pick an answer choice consistent with what we expect when people make attributions for their own behavior. 

Attributions are classified as either internal or external. Internal attributions include dispositional or personality-based explanations; external attributions emphasize situational factors. The passage tells us the participants showed a self-serving bias. The self-serving bias refers to the tendency to attribute internal factors for success and external factors for failure, particularly when someone is explaining their own behavior. 

The author tells us in Paragraph 3, “When participants made in-group judgments and the behavior outcomes were negative, participants mainly attributed the behavior to situational factors. When participants made out-group judgments and the outcomes were positive, participants mainly attributed the behavior to situational factors.”

  1. People attribute positive and negative outcomes to dispositional factors. This is only partially correct, we do expect people attribute positive outcomes to dispositional factors, but we said people will attribute negative outcomes to situational factors.
  2. People attribute positive and negative outcomes to situational factors. Similar to answer choice A, this is only partially correct. We expect people attribute negative outcomes to situational factors, but people attribute positive outcomes to dispositional factors.
  3. People attribute positive outcomes to dispositional factors and negative outcomes to situational factors. This answer choice is consistent with our breakdown. People have the tendency to attribute internal factors for success and external factors for failure. People attribute positive outcomes to dispositional factors and negative outcomes to situational factors. This is our best answer choice.
  4. People attribute positive outcomes to situational factors and negative outcomes to dispositional factors. This is the opposite of our breakdown and what we learned from the study. People attribute positive outcomes to dispositional factors and negative outcomes to situational factors. We can stick with answer choice C as our best option.

16) This is a similar question to others we’ve seen in this question set which makes sense because we’re focused on the findings of the study in the passage. We’re given a hypothetical in the question stem that talks about team-building. If the consultant’s team-building activities are successful, we would most likely see one of the four answer choices based on what we read in the passage. Recall that people have the tendency to attribute internal factors for success and external factors for failure. In other words, people attribute positive outcomes to dispositional factors and negative outcomes to situational factors. 

We saw in the passage that group membership can influence how people attribute positive and negative outcomes. The consultant would ideally break down any barriers and get the managers and assistants to feel they are part of the same team; managers and assistants would more often attribute positive outcomes to dispositional factors and negative outcomes to situational factors.

  1. Managers would attribute positive outcomes by other managers to situational factors. This is the opposite of what we would expect. People attribute positive outcomes to dispositional factors, particularly within their group. As they go through team-building exercise, we would expect managers would attribute positive outcomes to dispositional factors.
  2. Managers and assistants would attribute positive outcomes by managers to dispositional factors. This answer choice is consistent with our breakdown of the question. The consultant would ideally get the managers and assistants to feel they are part of the same team and more often attribute positive outcomes to dispositional factors and negative outcomes to situational factors. 
  3. Assistants would attribute negative outcomes by other assistants to dispositional factors. This is the opposite of what we would expect after going through team-building activities. Assistants are already likely to attribute negative outcomes by other assistants to situational factors, so we expect that would continue or happen even more.
  4. Managers and assistants would attribute negative outcomes by managers to dispositional factors. This is also the opposite of what we would expect. Managers and assistants would feel closer due to the team-building activities and would therefore attribute negative outcomes by managers to situational factors. We can stick with our best answer choice here, answer choice B. 

Exam 4 P/S Solutions: Passage 4

17) This is a passage-based question that relies on us going back to the passage to pick out some key information. The author tells us in Paragraph 3 that these programs are designed to enhance socialization for school-aged children and some include the added goal of increasing mindfulness skills and strategies. 

Piaget focused on children’s cognitive growth. His theory of cognitive development holds that our cognitive abilities develop through specific stages. Piaget proposed four stages of cognitive development: the sensorimotor, preoperational, concrete operational and formal operational period. We’re going to decide who will benefit the most from the programs described in the passage according to Piaget’s theory of cognitive development.

  1. sensorimotor individuals, since they have the capacity for abstract thought. The sensorimotor stage is the first of the four stages in cognitive development which extends from birth to the acquisition of language. In this stage, infants progressively construct knowledge and understanding of the world by coordinating experiences with physical interactions with objects. Abstract thought is associated with the formal operational stage and happens later in life. This is a factually incorrect answer choice.
  2. preoperational individuals, since they are imaginative and vulnerable to influence. The preoperational stage starts when the child begins to learn to speak at age two and lasts up until the age of seven. In this stage, children do not yet understand concrete logic and cannot mentally manipulate information. The child can form stable concepts, but perspectives are limited by egocentrism. This is the age where the programs would have a great benefit. The children are vulnerable to influence so the enhanced socialization, increased mindfulness, and strategies will be beneficial.
  3. phallic individuals, since they have the ability to regulate desire more fluidly. This question is focused on Piaget’s theory, not Freud’s five stages of development. This answer choice incorrectly lists the phallic stage of development which is inconsistent with Piaget’s theory.
  4. latent individuals, since they are unthwarted by sexual desire and are more focused. Similar and answer choice C, this answer choice incorrectly lists the latency stage of development (Freud) which is inconsistent with Piaget’s theory. We can stick with answer choice B as our best answer. 

18) Similar to the last question in this set, this is a passage-based question that relies on us going back to the passage to pick out key information. The author talks about the difficulties in implementing programs to help with social skills training in Paragraph 5: 

The author introduces why some community members might be resistant to change, and also explains how their perspectives may change. We’re specifically told, “Critics of social skills training programs might be swayed if the trainers work pro bono, purely for the joy of enacting change and helping others.” We’re also told additional scientific evidence may sway perspectives. This paragraph is going to be key to answering our question. We can go through our four choices, define each one, and then find the answer choice that best answers our question. 

  1. attraction. Trainers acting out of attraction might imply the trainers should get critics of social skills training programs to like them more and possibly build better relationships. This was not discussed in the passage as a possible way program implementation succeeds. 
  2. aggression. Aggression describes a range of behaviors that are intended to cause harm to others; aggression may be physical, mental, or verbal, and varies according to cultural factors. This would likely hurt the chances of having a program implemented.
  3. attachment. Trainers acting out of attachment is similar to answer choice A. Attachment, in this case, could be a strong bonding toward or with the trainers which is not discussed in the passage. Attachment can also be between a child and a parent, but this is also not discussed as a way by which program implementation will succeed.
  4. altruism. Altruism is the desire to help others even if the costs outweigh the benefits of helping. This is exactly what the author suggests when saying, “Critics of social skills training programs might be swayed if the trainers work pro bono, purely for the joy of enacting change and helping others.” By working for free, the trainers show they are working to help others and this would sway some critics of the programs.

19) This is a fairly typical question setup for the behavioral section of the exam. We’re asked about something in the passage from a specific perspective or school of thought. That means we not only have to have a good understanding of what we read in the passage, but also a mastery of the vocabulary in this section. 

This question asks about the sociological perspective of labeling theory. Labeling theory concerns itself not with the normal roles that define our lives, but with those very special roles that society provides for deviant behavior, called deviant roles, stigmatic roles, or social stigma. Labeling theory hypothesizes that the labels applied to individuals influence their behavior; the application of negative or stigmatizing labels promotes deviant behavior. 

  1. social stigma applied to some students. We talked about this in our breakdown of the question. Labeling theory hypothesizes that the labels applied to individuals influence their behavior. The use of stigmatizing labels that promote deviant behavior can be counteracted with social skills training. This is a strong answer choice. 
  2. social capital represented by the school. Social capital includes the advantages conferred by one’s social network, such as access to professional opportunities and insider knowledge. These types of capital facilitate mobility by providing access to opportunities and the tools to acquire wealth and status. This answer choice is out of scope as we’re focused on labeling theory and the students, not so much the school. 
  3. cultural capital associated with the school. Cultural capital is the accumulation of knowledge, behaviors, and skills that a person can tap into to demonstrate one’s cultural competence and social status. Much like answer choice B, this answer choice is out of scope as we’re focused on labeling theory and students, not so much the school.
  4. cultural diversity reflected in the students. Cultural diversity has to do with different (diverse) cultures. Reasoning here is going to be the same as answer choices B and C; the answer is out of scope as we’re focused on labeling theory. We can stick with answer choice A as our best answer.

20) This is another passage-based question but the author explicitly tells us we’re answering this question based on what we read in Paragraph 3:

The author tells us in Paragraph 3 that these programs are designed to enhance socialization for school-aged children and some include the added goal of increasing mindfulness skills and strategies. Socialization is the process by which the new generation learns the knowledge, attitudes and values that they will need as productive citizens. To answer this question correctly, we’ll find the answer choice that is consistent with Paragraph 3 and our definition of socialization.

  1. show students how to reconcile conflicts across social roles. This is describing role conflict. Role conflict describes the conflict between or among the roles corresponding to two or more statuses held by one individual. We’re looking for a design consistent with socialization. 
  2. promote the integration of students across diverse cultures. This answer choice is describing multiculturalism which is the practice of valuing and respecting differences in culture. We’re looking for a design consistent with socialization.
  3. encourage new students to assimilate to the school culture. Assimilation is the process by which an individual or group becomes part of a new culture. Our reasoning here is going to be similar to the reasoning we used for answer choices A and B–we’re looking for a design consistent with socialization.
  4. enhance students’ learning of school norms and peer values. This answer choice is consistent with Paragraph 3 and the definition of socialization. The programs are designed to enhance socialization which involves learning of school norms and peer values. Answer choice D is our best choice.

Exam 4 P/S Solutions: Passage 5

21) This is a passage-based question, but the test-maker explicitly tells us where in the passage we’re looking for the necessary information. In the first sentence of the passage, the author says, “Early studies have shown that the less attraction individuals feel toward a group to which they belong, the less their opinions are affected by the group.” My first impression is this is a direct correlation because the less (or more) attraction individuals feel toward a group to which they belong, the less (or more) their opinions are affected by the group. As one variable increases or decreases, the other variable also increases or decreases accordingly.

  1. A positive correlation. A positive correlation indicates that as one variable changes, the other variable changes in the same direction. For example, we would say that weight and height are positively correlated because one tends to increase as the other increases. A positive/direct correlation is exactly what we said in our breakdown of the question and what the author tells us in the first sentence of the passage. This is a strong answer to start. 
  2. A negative correlation. A negative correlation indicates that as one variable changes, the other variable moves in the opposite direction. This is the opposite of what the author says in the first part of the passage. Answer choice A remains superior.
  3. An inverse effect. An inverse effect would imply something similar to what we said in answer choice B, but causation. An inverse effect implies the individual’s attraction to the group causes a decrease in the group’s influence on an individual’s opinion (or vice versa). This is not what consistent with the relationship we’re told about in the beginning of the passage.
  4. A reverse effect. This is similar to answer choice C because a reverse effect would imply causation. We don’t know if an individual’s attraction to the group causes a change in the group’s influence on that individual’s opinion (or vice versa). This is not what consistent with the relationship we’re told about in the beginning of the passage. Answer choice A is going to be our best answer choice.

22) To answer this question, we can consider where the author talks about ranking of the attributes. We’re told, “Participants were placed in discussion groups. After socializing, participants were presented with a list of attributes and asked to privately rank them in order of importance.” 

In this question, we’re jumping into a hypothetical situation in which participants discuss their ranking of the attributes with members of their group and found they agreed. Following this agreement, we want to identify the psychological process that is most likely to affect each participant’s opinion. We can define each of the four choices and pick the most likely answer. 

  1. Reaction formation. Reaction formation is a defense mechanism. In reaction formation, individuals will have beliefs and emotions that they feel are unacceptable, or that are anxiety-inducing. The individual will instead act to do the opposite to counteract these negative feelings. This is not what would happen to the participants.
  2. Social loafing. Social loafing is the tendency for people to put forth less effort when working on a group task if the individual contributions aren’t evaluated. The participants aren’t working on a group task here, but rather they are having a discussion.
  3. Selective attention. Selective attention is the ability to focus on individual stimuli whilst ignoring irrelevant stimuli in the subject’s perception. This is out of scope here while discussing the group discussions.
  4. Group polarization. Group polarization is the tendency for groups that are initially in agreement to make decisions that are more extreme than the initial inclination of its members. Group polarization most likely would affect the opinions of the participants. Answer choice D is our best option. 

23) The author introduces the ranking of attributes in Paragraph 2 and gets into the bogus group consensus in Paragraph 3. The rankings and bogus group consensus will make the participants feel as though their group is defined as one of the four group types given as answer choices. We can define the four groups given as options and find the one that best answers our question. 

  1. a reference group, so that participants will evaluate themselves against the group. A reference group refers to a group to which an individual or another group is compared. In the passage, participants are told their rankings diverged from the group’s rankings, meaning we have comparison. Participants are ranking one another and researchers are seeing how others’ opinions might change the perspective of the participants. While this isn’t the perfect definition of a reference group, it’s a solid answer to start.
  2. a primary group, so that participants will want others in the group to be respected. A primary group is typically a small social group whose members share close, personal, enduring relationships. These groups are marked by a concern for one another, shared activities and culture, and long periods of time spent together. Based on what we know from the passage, the participants will not feel as though their group is a primary group.
  3. an in-group, so that participants will consider themselves to be similar to the group. An in-group is a social group to which an individual feels they belong as a member. Consider the background we’re given in Paragraph 3 and how the participants likely felt toward the group. Receiving the bogus group consensus indicating that the participants’ ranking diverged from the group’s ranking, then receiving the bonus acceptance rating indicating they were only marginally accepted will not make the participants feel like they belong to the group. The participants likely do not feel like they are similar to the group. Answer choice A remains a superior answer. 
  4. a status group, so that participants will want others in the group to view them favorably. A status group will contain members with similar standing in the community, lifestyle, and prestige. This is out of scope. We know the participants evaluated themselves against the group so we’re going to stick with answer choice A as the superior answer. The ranking of attributes and the bogus group consensus are best described as making participants feel as though their group is a status group.

24) This is a pseudo-standalone question that’s related to the topic presented in the passage, but we’ll answer using our external knowledge. Groups are collections of people who identify and interact with one another and are united in some way. We want to know when attraction and commitment to this group is greatest. That means we’ll need to know about group dynamics and how group dynamics can change. Cohesion in a group usually comes from shared interests, values, representations, ethnic or social background, and kinship ties, among other factors. With that in mind, we’ll attack this question similarly to how we attacked the previous questions in this set. We can define each of the four answer choices and find the one that best represents when attraction and commitment to a group is greatest. 

  1. promote their social networks. A social network is a social structure between individuals or organizations. Promoting social networks is not something that will likely increase each of the members’ attraction and commitment to the group. We’re instead looking for something along the lines of shared interests, values, representations, or similar factors.
  2. express diverse cultural values. The members of the group may or may not find this as a positive. Diverse cultural values might seem like a positive for a group, but we’re focused on the existing group members specifically. Groups are stronger when members have shared values rather than diverse values. 
  3. share equivalent cultural capital. I mentioned in the breakdown of the question: Cohesion in a group usually comes from shared interests, values, representations, ethnic or social background, and kinship ties, among other factors. Cultural capital is the accumulation of knowledge, behaviors, and skills that a person can tap into to demonstrate one’s cultural competence and social status. Shared equivalent cultural capital will lead to attraction and commitment to the group by its members. Groups are stronger when members share the most similarities. This is going to be our best answer choice.
  4. present their back-stage selves. The back-stage self is part of a theater metaphor. The back-stage self is employed when players are together, but no audience is present. That means players can let go of conventions and include behavior that’s unacceptable in front of an audience. When part of the group (or society in general), members will typically present their front-stage self. Answer choice C remains our best option here.

Exam 4 P/S Solutions: Questions 25-28

25) When asked whether a person who is afraid of spiders would be diagnosed as having a psychological disorder, a psychologist replies, “It depends on whether or not this fear interferes with the person’s life.” The psychologist appears to rely most heavily on which criterion of abnormality? This is a classic behavioral standalone question in that it relies on our external knowledge, specifically vocabulary. In order to classify behavior as abnormal, psychologists generally look at 4 criteria: violation of social norms, statistical rarity in the population, personal distress and maladaptiveness. We want to define each of our four answer choices and find the one that is consistent with psychological disorders interfering with one’s life.

  1. Distress. Distress has to do with negative feelings (like stress) or abnormal behavior that can be debilitating if it is continuous. This is inconsistent with what the psychologist replied to the person in the question stem.
  2. Maladaptiveness. Maladaptiveness means the behavior interferes with the person’s normal life, which is exactly what the psychologist in the question stem used to define a psychological disorder. This is a strong answer choice.
  3. Statistical deviancy. Data tend to fall within a certain level, but statistical deviancy, like the name suggests, is when we see deviation from this expected level. That is not consistent with what the psychologist replied and is not the best criterion of abnormality. Answer choice B remains the best option.
  4. Violation of social norms. Social norms are the explicit or implicit rules specifying what behaviors are acceptable within a society or group and include sanctions, folkways, mores, taboos, and anomie. Being afraid of spiders does not violate social norms and is inconsistent with the reply from the psychologist; this is not the appropriate criterion of abnormality within the context of this question. Answer choice B is going to be our best answer. 

26) This is another standalone question that relies on external knowledge, specifically vocabulary. Erikson’s eight stages of psychosocial development include trust vs mistrust, autonomy vs shame/doubt, initiative vs guilt, industry vs inferiority, identity vs role confusion, intimacy vs isolation, generativity vs stagnation, and integrity vs despair.

Generativity vs Stagnation: When people reach their 40s, they enter the time known as middle adulthood, which extends to the mid-60s. The social task of middle adulthood is generativity vs stagnation. Generativity involves finding your life’s work and contributing to the development of others through activities such as volunteering, mentoring, and raising children.

  1. adolescent. This corresponds to identity vs. role confusion. In adolescence (ages 12–18), children face the task of identity vs role confusion. According to Erikson, an adolescent’s main task is developing a sense of self. Adolescents struggle with questions such as “Who am I?” and “What do I want to do with my life?” Along the way, most adolescents try on many different selves to see which ones fit; they explore various roles and ideas, set goals, and attempt to discover their “adult” selves.
  2. young adult. This corresponds to intimacy vs. isolation. Intimacy vs Isolation: People in early adulthood (20s through early 40s) are concerned with intimacy vs isolation. After we have developed a sense of self in adolescence, we are ready to share our life with others.
  3. middle-aged adult. A middle-aged adult would most likely be struggling to achieve generativity rather than stagnation. This is our correct answer. 
  4. elderly adult. This corresponds to integrity vs despair. Integrity vs Despair: From the mid-60s to the end of life, we are in the period of development known as late adulthood. Erikson’s task at this stage is called integrity vs despair. He said that people in late adulthood reflect on their lives and feel either a sense of satisfaction or a sense of failure. Answer choice C is the best answer. 

27) A dopamine antagonist will most likely lead to: This is another standalone question that relies on external knowledge. Dopamine is a neurotransmitter that is involved in mood, sleep, and learning. It can also suppress appetite and increase pleasure receptors. An antagonist blocks or impedes the regular activity of a neurotransmitter at the receptor. We want to which of the four answer choices a dopamine antagonist will most likely lead to. Two of the answer choices deal with schizophrenia, while two deal with Parkinson’s. We can go through some background first:

Schizophrenia is a serious and often-debilitating mental illness. The development of schizophrenia is thought to involve malfunctioning dopaminergic neurons and may also involve problems with glutamate signaling. Treatment for the disease usually requires anti-psychotic medications that work by blocking dopamine receptors and decreasing dopamine neurotransmission in the brain. This decrease in dopamine can cause Parkinson’s disease-like symptoms in some patients. While some classes of anti-psychotics can be quite effective at treating the disease, they are not a cure; most patients must remain medicated for the rest of their lives.

Parkinson’s disease causes the loss of dopamine neurons in the substantia nigra, a midbrain structure that regulates movement. Loss of these neurons causes many symptoms including tremors (shaking of fingers or a limb), slowed movement, speech changes, balance and posture problems, and rigid muscles. The combination of these symptoms often causes a characteristic slow, hunched, shuffling walk. Patients with Parkinson’s disease can also exhibit psychological symptoms, such as dementia or emotional problems.

  1. increased hallucinations in patients with schizophrenia. Treatment for schizophrenia usually requires anti-psychotic medications that work by blocking dopamine receptors and decreasing dopamine neurotransmission in the brain. We expect a decrease in hallucinations. 
  2. increased delusions in patients with schizophrenia. Treatment for schizophrenia usually requires anti-psychotic medications that work by blocking dopamine receptors and decreasing dopamine neurotransmission in the brain. We expect a decrease in delusions.
  3. decreased mobility and decreased frequency of tremors in patients with Parkinson’s disease. A dopamine antagonist will decrease dopamine levels that are already too low. Although we expect decreased mobility, we would expect an increased frequency of tremors.
  4. decreased mobility and increased frequency of tremors in patients with Parkinson’s disease. A dopamine antagonist will decrease dopamine levels that are already too low. We expect decreased mobility and increased frequency of tremors. This is going to be our best answer choice.

28) This is a standalone question that is asking us to differentiate between mean and median age. Mean age is a numerical descriptor that’s synonymous with average, while median is synonymous with the middle value. When we have a big difference like we see in our median and mean ages, it’s likely there were a lot of subjects that were much older that skewed the mean higher. The median simply means there were the same number of subjects under the age of 45 than there were over the age of 45. Age 45 is the middle number of the number set.

  1. The sample had more subjects under the age of 45 than over the age of 45. Median is the “middle number” so the sample had the same number of subjects under the age of 45 and over the age of 45. 
  2. The sample had more subjects over the age of 45 than under the age of 45. Median is the “middle number” so the sample had the same number of subjects under the age of 45 and over the age of 45.
  3. The sample included subject(s) who were much older than the age of 45. This answer choice is correct. The mean was higher than the median because the sample included subjects that were much older than 45 and skewed the mean higher. 
  4. The sample included subject(s) who were much younger than the age of 45. This answer choice is incorrect. If the sample included subjects who were much younger than the age of 45, we would expect a lower mean. We can stick with answer choice C as our best option. 

Exam 4 P/S Solutions: Passage 6

29) This is a passage-based question that relies on us going back to the passage to pick out some key information. We can revisit Paragraph 4. The author says, “Exposure to violent media can also have long-term effects through learning.” This is consistent with our question stem. We’re going to get into the specific effects. The author says, “Specifically, repeated exposure can lead to the observation and reinforcement of aggressive behaviors, the storage of aggressive knowledge structures (such as aggressive expectations or aggressive behavior scripts), and losing responsiveness to aggression.” We’re making a connection between the long-term effects of exposure to violence and our four answer choices. We can briefly describe each one and pick the best answer choice.

  1. behavioral disorders. The author does mention that repeated exposure can lead to aggressive behaviors, there is no mention of causing behavioral disorders. 
  2. schemas. A schema is an organizing pattern of thought that is used to categorize and interpret information, thus shaping individual attitudes and perspectives. From these schema frameworks, specific behavioral scripts develop as plans for how to behave in certain situations. The author explicitly mentions aggressive behavior scripts following repeated exposure. This is going to be our best answer choice so far.
  3. motivations. Motivation is a psychological factor that provides a directional force or reason for behavior. Paragraph 4 listed several effects following repeated exposure to aggressive behaviors, but did not mention providing individuals with aggressive motivations. We can stick with answer choice B as our superior answer. 
  4. drives. Reasoning here will be similar to the reasoning in answer choice C. A drive is an urge to perform certain behaviors in order to resolve physiological arousal when that arousal is caused by the biological needs of the organism. Paragraph 4 listed several effects following repeated exposure to aggressive behaviors, but did not mention providing individuals with aggressive drives. Answer choice B is our correct answer.

30) The test-maker sets up this question by presenting us with a declarative statement: nonviolent video games can alter internal states by causing frustration. We talked about violent media in the passage, but little mention was made of video games specifically. However, given what we read in the passage, we want to explain how nonviolent video games differ from violent video games as situational variables. Remember, nonviolent video games can cause frustration and alter internal states (cognitive states, affective states, and physiological arousal). Three of our answers are going to be applicable to nonviolent video games, but our correct answer is something nonviolent games are NOT likely to do.

  1. influence physiological arousal. The key here is going to be the frustration caused by the nonviolent video games. Frustration can be related to physiological arousal and negative emotions may subsequently arise. Because nonviolent video games are likely to influence physiological arousal, this is an incorrect answer choice. 
  2. change affective states. Attitudes are thought to have three components: an affective component (feelings), a behavioral component, and a cognitive component. Frustration is an affective state, so this answer choice is incorrect.
  3. activate the nervous system. This ties into what we talked about in answer choices A and B. The frustration felt while playing the nonviolent video game will active the nervous system. The sympathetic nervous system plays a role in the emotional reaction following frustration. We can eliminate this answer choice.
  4. prime aggressive thought structures. We learned in Paragraph 3 that watching violence can directly and indirectly prime aggressive thoughts. Nonviolent video games, however, are not likely to prime aggressive thought structures. This is going to be our best answer choice. 

31) This is another passage-based question, but the author explicitly tells us we’re answering this question based on what we read in Paragraph 2. We’re going to revisit the example in that paragraph and find the theory of emotion that is most compatible with that example. That means we’ll be relying on our external knowledge to define the four theories of knowledge listed as answer choices.

Which statement best describes its usefulness in deciding whether a given situation is abnormal?

The example in Paragraph 2 is the classmate bumping into the child.

A. The James–Lange theory, because it suggests that physiological arousal triggers affective states. The James–Lange theory of emotion asserts that emotions arise as a result of physiological arousal —i.e., that the self-perception of changes in the body produces an emotional experience.

Which statement best describes its usefulness in deciding whether a given situation is abnormal?

B. The Cannon–Bard theory, because it suggests that physiological arousal influences affective states. Cannon–Bard theory argues that physiological arousal and emotional experience occur simultaneously, yet independently. According to the Cannon–Bard theory, when you see a venomous snake, you feel fear at exactly the same time that your autonomic nervous system responds.

Which statement best describes its usefulness in deciding whether a given situation is abnormal?

C. The Schachter–Singer theory, because it focuses on the role of cognitive appraisal in influencing affective states. According to the Schacter–Singer theory, emotion results from the interaction between two factors: physiological arousal and cognition. More specifically, this theory claims that physiological arousal is cognitively interpreted within the context of each situation, which ultimately produces the emotional experience. These cognitive interpretations, how a person labels and understands what they are experiencing, are formed based on the person’s past experiences. The child being bumped into involves cognitive interpretation. This is going to be our best answer choice.

Which statement best describes its usefulness in deciding whether a given situation is abnormal?

D. The evolutionary theory, because it focuses on the adaptive functions of affective states in response to threats from the environment. The evolutionary theory of emotion essentially says emotions serve an adaptive role in responses to threats from the environment. This is out of scope in the context of this question as Paragraph 2 does not reference the adaptive functions of affective states.

32) This is a passage-based question that relies on us going back to the passage to pick out some key information. We can revisit Paragraph 4. The author says, “repeated exposure can lead to the observation and reinforcement of aggressive behaviors, the storage of aggressive knowledge structures (such as aggressive expectations or aggressive behavior scripts), and losing responsiveness to aggression.” We’re going to relate the four types of learning given as answer choices to this information from Paragraph 4.

A. operant conditioning. Operant conditioning is a theory of learning that focuses on changes in an individual’s observable behaviors. In operant conditioning, new or continued behaviors are impacted by new or continued consequences. Reinforcement and punishment are principles of operant conditioning that increase or decrease the likelihood of a behavior. The passage explicitly mentions reinforcement of behaviors.

Which statement best describes its usefulness in deciding whether a given situation is abnormal?

B. social learning. Social learning is a cognitive process that takes place in a social context and can occur purely through observation or direct instruction. The author mentions observation of aggressive behaviors to explain the long-term effects of exposure to media violence.

C. classical conditioning. Classical conditioning is learning in which the stimulus or experience occurs before the behavior and then gets paired or associated with the behavior. The author does not refer to classical conditioning in the passage. This is going to be our best answer.

D. habituation. Habituation is the diminishing of a physiological or emotional response to a frequently repeated stimulus. The author references losing responsiveness to aggression. This is an incorrect answer choice. The passage makes a reference to all of the listed types of learning except answer choice C: classical conditioning.

Exam 4 P/S Solutions: Passage 7

33) The author tells us that researchers hypothesized that traditional gender attitudes increase alcohol consumption in young men, while such attitudes reduce consumption in young women. We want to explain how gender socialization affects drinking behavior. This is a pseudo-standalone question that’s related to the topic presented in the passage, but we’ll answer using our external knowledge. 

Gender socialization is the specific process of learning the cultural norms associated with gender roles, masculinity, and femininity. We’re ultimately looking for an answer that talks about how men and women learn norms and values. Norms can be defined as the shared ways of thinking, feeling, desiring, deciding, and acting, which is observable in regularly repeated behaviors and is adopted because they are assumed to solve problems. Values are culturally-defined standards that serve as broad guidelines for social living. 

  1. Young men learn norms that encourage high-risk behaviors. Young men learn norms like risk-raking, emotional restraint, and self-reliance. Drinking (and drinking heavily) is an example of this encouraged high-risk behavior. This is a strong answer choice.
  2. Young men learn values that emphasize cooperative behaviors. Cooperative efforts will involve working together to maximize the collective outcome. Young men are taught to compete and win against others, especially their peers. Values emphasizing cooperative behaviors are not affecting drinking behavior.
  3. Young women learn norms that encourage high-risk behaviors. Young women are not encouraged to partake in high-risk behaviors. That is seen more in young men. Young women learn norms that are associated with femininity. Answer choice A remains superior.
  4. Young women learn values that emphasize cooperative behaviors. While this statement may be true in a vacuum, it does not provide an explanation for how gender socialization affects drinking behavior. Values emphasizing cooperative behaviors are not affecting drinking behavior. Answer choice A is our correct answer.

34) This question is asking about one of the selected variables in the research. We’re given data about the gender role attitude scale, but we’re asked to decide which component of attitude is being measured. The author covers this in Paragraph 2. Specifically, the author says, “Gender role attitudes were measured by coding respondents’ level of agreement or disagreement with statements that assert beliefs about gender and family life, resulting in scores for each respondent on a gender role attitude scale.”

Attitude is our evaluation of a person, an idea, or an object. Typically, attitudes are favorable or unfavorable, or positive or negative. They can also be defined as a learned habit for responding to social stimuli. Attitudes form from three components; the affective, behavioral and cognitive.

The affective component of attitude relates to a person’s feelings or emotions in their shaping on attitudes to a person or object. If you feel more positive about someone you are more likely to address them in a positive manner.

The cognitive component relates to our beliefs and knowledge about someone or a situation that shapes our attitude. For example, if you know dogs bite and this is dangerous you would adapt your attitude to dogs when you see them because of your knowledge of them.

The final component is behavioral. This component involves our actions towards a person or situation. For example, if we have a positive behavior at work and there is a positive environment we are more likely to behave in a productive manner.

  1. Persuasive. This answer choice is out of scope. Attitudes form from three components; the affective, behavioral and cognitive.
  2. Behavioral. The behavioral component involves our actions towards a person or situation. We’re focused on beliefs about the person or object instead.
  3. Cognitive. The cognitive component relates to our beliefs and knowledge about someone or a situation that shapes our attitude. This matches what we’re looking for exactly and is consistent with what the author says in the passage. Gender role attitudes were measured by coding respondents’ level of agreement or disagreement with statements that assert beliefs about gender and family life. This is going to be our best answer. 
  4. Affective. The affective component relates to a person’s feelings or emotions in their shaping on attitudes to a person or object. We’re instead focused on beliefs about the object, not feelings/emotions. Answer choice C is our best answer choice. 

35) This is a passage that revolves around research, so the test-maker decided to ask us about research designs. The key here is we’re comparing alcohol consumption between groups. In the passage, the author tells us marriage was associated with reduced alcohol consumption for both genders. Researchers compared alcohol consumption for single males and females versus their married counterparts. Researchers would have to utilize a design along the same lines to determine whether the findings hold for same-sex marriage as well. We can go through our four answer choices and see which one would be the best design.

  1. Ask a random sample of respondents whether individuals in same-sex marriages consume alcohol in different patterns than married heterosexuals. This answer choice brings up a research design that has to do with opinion instead of instead of quantitative results about alcohol consumption. 
  2. Conduct a study of gender role attitudes that consists of respondents currently living with a partner, including those in same-sex marriages. While this information might be helpful, this research design does not tell us about alcohol consumption in the different groups. Alcohol consumption is a key variable we’re looking at. 
  3. Survey alcohol consumption patterns of a random sample of American adults across demographic characteristics, including sexual orientation. We’re looking for a research design that measures the role of same-sex marriages directly. This would, again, be useful information as a supplement to the research, but does not answer the specific question being asked or fill the specific role researchers need from their research design. 
  4. Compare rates of alcohol consumption among a random sample of single, homosexual respondents and those married to a same-sex partner. This is actually what researchers did in the initial study. Researchers compared alcohol consumption for single males and females versus their married counterparts. This design would allow researchers to determine whether the studies hold for same-sex marriage as well. This design considers alcohol consumption (unlike answer choice B) as well as the role of same-sex marriages (unlike answer choice C). This is going to be our best answer choice.

36) This is another passage-related question, but this is a pseudo-standalone question. The question is related to the topic presented in the passage, but we’ll answer using our external knowledge and what we know about research design. Best way to answer this question is go through each of the research designs listed as possible answer choices and define each one. We will find the one that best describes the study in the question stem.

  1. cross-sectional. Cross-sectional research describes study done at a specific point in time. By following the respondents every 2 years for a decade, researchers are no longer utilizing a cross-sectional research design.
  2. experimental. Experimental research is research in which scientists manipulate variables to test the difference between an experimental and control group. This is inconsistent with the method described in the question stem.
  3. longitudinal. Longitudinal research describes a research study that follows the population or group of interest at multiple, long time points. That’s exactly what the researchers are doing in the question stem. This is going to be our best answer.
  4. historical. Historical research is focused on the past using primary or archival sources. The research design in the question stem is not historical research. Answer choice C is our best answer. 

37) This is going to be similar to some of the other questions in this set because we’re asked about something the author tells us in the passage, but we’ll use external knowledge to actually answer the question being asked. We’re told in Paragraph 1, “Research has found gender differences in alcohol consumption among adolescents and young adults. Compared to young women, young men report drinking more often, as well as consuming more alcohol on occasions when they drink.” We’re going to go through the four options presented by the test-maker and see which ones can provide a potential hypothesis for the gender difference in alcohol consumption between young men and women. Our correct answer will not provide a potential hypothesis for the difference.

  1. Co-morbid psychological disorders will be more likely to increase alcohol consumption for young men than young women. This is a potential hypothesis. Certain psychological disorders can be related to increased alcohol consumption. If men are at higher risk for a specific disorder, that can also mean they are more likely to increase alcohol consumption.
  2. The number of dependents living at home will be higher for young men than young women. This might be valuable information for a different study, but it does not relate to alcohol consumption. We want a potential hypothesis for the gender difference in alcohol consumption between young men and women, but this answer choice simply tells us about the number of dependents living at home. This is a strong answer choice.
  3. Gender identity will have a stronger effect on alcohol consumption for young men than young women. The author actually touches on gender identity quite a bit in the passage. If gender identity has a strong effect on alcohol consumption for young men, then that would explain the gender difference in alcohol consumption. Answer choice B remains the best option.
  4. Alcohol consumption will be more likely to function as a coping strategy for young men than young women. This is once again a potential hypothesis for the gender difference in alcohol consumption between young men and young women. The hypothesis is related to alcohol consumption and, most importantly, provides a potential reason for the difference in alcohol consumption. Only answer choice B does not provide a potential hypothesis for the gender difference in alcohol consumption between young men and young women.

38) The test-maker provides a possible relationship between alcohol consumption in men and higher testosterone levels. To answer this question, we may need to use Table 1 from the passage that showed the results of the study, so we can pull it up for reference. 

We’re focused on alcohol consumption and testosterone levels, but our correct answer will ultimately cast doubt about the hypothesis that higher alcohol consumption in men, compared to women, may be related to higher testosterone levels in men.

  1. Employment increases alcohol consumption among men but decreases it among women. This answer choice correctly talks about differences in alcohol consumption, but it relates these differences to employment. We do not know if testosterone is playing a direct role here, but it is showing that there is increased alcohol consumption in men. This does not cast doubt on the hypothesis.
  2. Traditional gender attitudes are linked to a similar reduction in drinking for men and women. This is a better option than answer choice A, but still vague. There’s a similar reduction in drinking for men and women, but that doesn’t change the fact there was a discrepancy in alcohol consumed to start. A similar reduction can still mean men drink more than women and this does not cast doubt on the hypothesis. We can eliminate answer choice A for now and see if we come across any better options than answer choice B. 
  3. Women consume less alcohol than men regardless of marital status and employment status. This would actually be the opposite of what we’re looking for. If we eliminate two factors like marital status and employment status but alcohol consumption is higher in men, then there’s not a higher chance that a factor like difference in testosterone is the driving force behind that difference. Answer choice B remains superior. 
  4. Men and women with similar attitudes and adult roles consume the same amount of alcohol. This answer choice sounds like what we’re looking for. We’re told that men and women with similar attitudes and adult roles consume the same amount of alcohol. That means differences in attitudes and roles were causing the discrepancy in consumption levels previously. Additionally, we’re essentially removing two factors (attitudes, roles) and find there is now similar consumption of alcohol, despite men having higher testosterone levels. That casts doubt on the hypothesis that higher alcohol consumption in men may be related to higher testosterone levels in men. Answer choice D is our best option. 

Exam 4 P/S Solutions: Passage 8

39) In this question, we’re asked about something the author tells us when covering Study 1, but we’ll use external knowledge to actually answer the question being asked. We’re focused on different types of information that are retrieved from memory. The author says, “The health history asked participants to report physical illnesses, such as asthma, or psychological disorders, such as depression, that they experienced before the age of 18.” We can go through and define the different memory types listed as our potential answer choices. We want to pick the one that is consistent with what the participants are asked to do in Study 1. 

  1. Episodic. Episodic memory is a kind of explicit memory, that includes specific events/episodes. When considering illnesses and disorders, the participants have to rely on episodic memory. They are retrieving personal memories of specific events/episodes. This is a strong answer choice.
  2. Semantic. Semantic memories are explicit memories that are not drawn from personal experience. These are common knowledge or concepts, but not as much autobiographic information. Answer choice A remains superior.
  3. Procedural. Procedural memories are a type of implicit memory, so they are less conscious. Like the name suggests, procedural memory involves being able to do tasks or procedures without conscious awareness of past experiences. Riding a bike and emotional conditioning are the classic examples. This is not used used when recalling childhood health history.
  4. Implicit. Implicit memories are long-term memories that are not consciously remembered, including procedural memories and emotional conditioning. Implicit memories do not involve recall like the participants were asked to do in Study 1. We can stick with answer choice A as our best option. 

40) This is a pseudo-standalone question that’s related to the topic presented in the passage, but we’ll answer using our external knowledge. The author mentions, “Statistical analyses revealed that retrospective reports of depressive symptoms or alcohol dependence were associated with lower income during adulthood.” We essentially want to pick the best way to assess alcohol use from the four options listed.

  1. Presence of aggressive behaviors. Aggressive behaviors can be a sign of alcohol use, among other things. While this is not specific to alcohol use, this is a strong answer choice to start.
  2. Presence of withdrawal symptoms. Similar to answer choice A, alcohol withdrawal symptoms are a sign of alcohol use. The difference between answer choice A and B, however, is answer choice B is the better answer choice for this specific question. While the presence of aggressive behaviors can be the result of many factors, alcohol withdrawal symptoms are the result of alcohol use. This is going to be our best answer choice so far. 
  3. Difficulty with memory function. This answer choice is similar to answer choice A. We do expect difficulty with memory function if there was alcohol use in the past, but difficulty with memory function can be the result of multiple factors and not just alcohol use and abuse. Answer choice B remains the best answer choice.
  4. Difficulty with impulse control. Reasoning here is going to be the same as answer choices A and C. Difficulty with impulse control can show up following alcohol dependence, but there are other factors that can lead to difficulty with impulse control besides alcohol dependence. We can stick with answer choice B as our best answer choice here. Withdrawal symptoms would be the best indicator of alcohol use and dependence. 

41) To answer this question, we can go back to Study 2 in the passage and note what the author says about childhood psychological disorders. The author says, “childhood psychological disorders predicted marital instability and were associated with the Big Five personality traits during adulthood.” The Big Five model distills personality traits down to 5 major traits: openness to experience, contentiousness (ie planning, responsibility), extroversion, agreeableness, and neuroticism (ie insecurity, anxiety). We want to find the characteristic that is least likely associated with childhood psychological disorders, or the characteristic that is not one of the 5 major traits. We can quickly glance at our answer choices and find the one that does not fall under openness to new experiences, contentiousness, extroversion, agreeableness, and neuroticism. The only answer choice listed that is not part of this list is answer choice D: Impulsivity. 

42) To answer this question, we can go back to Study 1 in the passage and note what the author says about risk factors for low SES in adulthood. The author says, “Statistical analyses revealed that retrospective reports of depressive symptoms or alcohol dependence were associated with lower income during adulthood.” These are the indicators for lower SES in adulthood that the author mentions in the passage, so we’re looking for an answer choice that is a symptom associated with depressive symptoms or alcohol dependence. 

  1. Repetitive behaviors. This is describing a characteristic of obsessive-compulsive disorder (OCD), which is a mental disorder characterized by intrusive thoughts and cognitions. These thoughts produce uneasiness, apprehension, fear, or worry, and by repetitive behaviors or rituals aimed at reducing the associated anxiety. Repetitive behaviors are not associated with depressive symptoms or alcohol dependence.
  2. Unwanted cognitions. This is going to be the same reasoning as answer choice A. Unwanted cognitions describes a characteristic of obsessive-compulsive disorder. Unwanted cognitions are not associated with depressive symptoms or alcohol dependence.
  3. Delusions of grandeur. This is a characteristic of schizophrenia. Schizophrenia is a disorder of psychosis in which the person’s thoughts, perceptions, and behaviors are out of contact with reality. Delusions of grandeur are not associated with depressive symptoms or alcohol dependence.
  4. Feelings of worthlessness. This is a characteristic of depression. Depression is a mood disorder characterized by a pervasive and persistent low mood that is accompanied by low self-esteem and worthlessness, and by a loss of interest or pleasure in normally enjoyable activities. A feeling of worthlessness is the symptom in childhood that is most likely to be a risk factor for lower SES in adulthood. Answer choice D is our best answer choice. 

Exam 4 P/S Solutions: Questions 43-46

43) This is a standalone question that relies on using our external knowledge to find a key term to describe the situation in the question stem. The test-maker provides us with a great visual in the question stem of a two-year-old using the same term (butterfly) for several objects that look similar. We’re going to go through our four answer choices and define each one. We want to find the one that best describes the situation in the question stem.

  1. the naming explosion. Naming explosion has to do with an increase in infant vocabulary. The term “explosion” is describing the rate with which the child is adding words to their vocabulary. While the two-year-old in the question stem is the appropriate age where we’d see the naming explosion, using the same term for objects that look similar is not a sign of the naming explosion. 
  2. overextension. Overextension is the application of a categorical definition to subjects outside of the actual category, usually because they look similar. This is exactly what we’re seeing in the question stem. Despite seeing moths and hair bows instead of actual butterflies, the child is calling similar-looking objects “Butterfly!” This is a great example of overextension. 
  3. categorical perception. Humans perceive things based on categories we have formed (hence the name categorical perception!). For example, when we see a rainbow, we see distinct, categorized colors. We don’t differentiate between the blue closer to the green color of the rainbow versus the blue closer to the indigo color of the rainbow. We just categorize the entirety of it as “blue” because that’s a category we have formed. This categorizing is prevalent when we have gradual changes, like the colors of the rainbow or in speech sounds, not categorizing objects as the same word because they look similar. Answer choice B remains the best option. 
  4. bootstrapping. This is the theory that children will learn word meanings using syntactic categories and language structure. Once children gain insight into this structure, they can make inferences about word meanings. Categorizing words as nouns, verbs, adjectives, etc. can help learn words, but is not what is happening when the two-year-old refers to similar-looking objects as “Butterfly!” Answer choice B remains our best answer. 

44) This is a standalone question that’s similar to a lot of the questions in the behavioral section of the exam. We’re going to rely on knowing vocabulary to get to the right answer. The test-maker provides us with a situation: teaching a dog to shake hands, and we want to pick one of the four answer choices listed as the most effective procedure by which to teach the dog. We can start by defining the vocabulary term in each answer choice and picking the best answer.

  1. Systematic desensitization. Systematic desensitization is a method by which people overcome phobias and anxiety. These people will learn relaxation or coping techniques and use these techniques to cope with their phobias and overcome their fears. This is not an effective procedure for teaching a dog to shake hands. 
  2. Elaborative encoding. Elaborative encoding uses information that is already known and relates it to the new information being experienced. The nature of a new memory becomes dependent as much on previous information as it does on the new information. This is not an effective procedure for teaching a dog to shake hands; there is no previous information on which the dog is building this new skill. 
  3. Shaping. Shaping involves a calculated reinforcement of a “target behavior”: it uses operant conditioning principles to train a subject by rewarding proper behavior and discouraging improper behavior. Shaping requires that the subject perform behaviors that at first merely resemble the target behavior; through reinforcement, these behaviors are gradually changed or “shaped” to encourage the target behavior itself. This describes exactly what happens when teaching a dog to shake hands. Through reinforcement, the dog will eventually shake hands properly.
  4. Spontaneous recovery. Spontaneous recovery is the return of a previously extinguished conditioned response. This is not an effective procedure for teaching a dog to shake hands because the dog is learning to shake for the first time. We can stick with answer choice C as our best answer choice. 

45) Just like our last question in this set, this is a standalone question that’s similar to a lot of the questions in the behavioral section of the exam. We’re relying on knowing vocabulary to get to the right answer. Depth cues are features of an object that are used by the visual system in the brain to interpret where an object is located in 3-D space. There are some depth cues that are monocular (meaning they require one eye). These include the retinal height of an object (how high up it is in the visual field), whether there is something occluding the object, and how textured an object appears. Three of our answer choices will be monocular depth cues, while our correct answer will not describe a monocular depth cue.

  1. Objects that are higher up in the visual field are perceived as being farther away than the objects that are lower in the visual field. This answer choice lists one of the depth cues we mentioned are monocular in our initial breakdown of the question.
  2. Objects that are occluded by other objects are perceived as being farther away than the objects that occlude them. Similar to answer choice A, this answer choice lists one of the depth cues we mentioned are monocular in our initial breakdown of the question.
  3. Objects that have more detailed textures are perceived as being closer than objects that have less detailed textures. Similar to answer choices A and B, this answer choice lists one of the depth cues we mentioned are monocular in our initial breakdown of the question
  4. Objects that are to the front of the point of focus are perceived as being closer than objects that are behind the point of focus. Retinal disparity is a binocular depth cue, meaning it requires both eyes. Retinal disparity refers to the fact that each of your eyes receives slightly different information about an object – your brain then uses this disparity to construct a perception of the object’s location in 3-D space. The distance of an object relative to the point of focus is associated with retinal disparity. Answer choice D is going to be our best answer choice.

46) Just like our last question in this set, this is a standalone question that’s similar to a lot of the questions in the behavioral section of the exam. We’re relying on our external knowledge, specifically about the amygdala. The amygdala is part of the limbic system and plays a role in processing emotional information and sending that information on to cortical structures. It is an important brain region in responding to fear and research suggests that the amygdala is involved in mood and anxiety disorders. Changes in amygdala structure and function have been found in adolescents who either are at risk for or have been diagnosed with a mood or anxiety disorder. We want an answer choice that is related to emotions like fear.

  1. Images designed to test color vision. We mentioned in our breakdown, the amygdala is part of the limbic system and plays a role in processing emotional information. The occipital lobe controls vision.
  2. A set of mathematical solving problems. Reasoning here is the same as answer choice A. The amygdala is part of the limbic system and plays a role in processing emotional information. When we think of logic or math problems, we’re referring to the left hemisphere of the brain, not necessarily the amygdala. 
  3. Images designed to evoke a fear response. This answer choice is consistent with our breakdown. We said the amygdala is an important brain region in responding to fear and research suggests that the amygdala is involved in mood and anxiety disorders. Images designed to evoke a fear response are likely to be included in a neuroimaging study to examine the function of the amygdala. This is going to be our best answer choice. 
  4. A set of reading comprehension passages on the topic of audition. Reading comprehension is not an effective way to look into the amygdala which plays a role in processing emotional information. Only answer choice C correctly lists an answer choice related to processing emotional information and responding to fear. Answer choice C is our best answer choice.

Exam 4 P/S Solutions: Passage 9

47) This is a passage-related question that focuses on three disorders discussed in the passage. In Paragraph 1, the author introduces depression, dementia, and alcohol use disorders. Underuse of available healthcare services for depression, dementia, and alcohol use disorders is likely due to one of the four vocabulary terms listed as answer choices here. We can go through each term and find the one that explains the underuse of available healthcare services for these disorders. 

  1. anomie. Anomie describes the alienation one feels from a lack of social norms, or the breakdown of social bonds between an individual and his community ties, resulting in the fragmentation of social identity. Anomie can lead to feelings of aimlessness or purposelessness, and occurs more frequently in periods of rapid social change. It is possible this explains the underuse of available healthcare services, but there was no emphasis on a lack of social norms in the passage.
  2. globalization. Globalization refers to the process of integrating governments, cultures, and financial markets through international trade into a single world market. This would actually provide the opposite effect as globalization would increase awareness about depression, dementia, and alcohol use disorders. We can eliminate this answer choice.
  3. culture lag. The term cultural lag refers to the notion that culture takes time to catch up with technological innovations, and that social problems and conflicts are caused by this lag. This is similar to answer choice A in that there was no talk of culture lag in the passage, nor does it apply to depression, dementia, and alcohol use disorders. We can hold on to answer choices A and C for now.
  4. social stigma. Stigma is the societal disapproval and judgment of a person or group of people because they do not fit their community’s social norms. The passage talks about educating the public about the symptoms of depression, raising public awareness of dementia, and attempting to change attitudes by disseminating factual information on the disease risks associated with drinking. The underuse of available healthcare services for depression, dementia, and alcohol use disorders is most likely due to social stigma. Answer choice D is the best answer.

48) This is a passage-based question that focuses on one of the three disorders discussed in the passage. We can focus on Paragraph 3 where the author talks about dementia.

The author mentions there is likely underreporting of dementia in LMICs and improving dementia surveillance can come in the form of raising public awareness through community-based programs. We want to get into some details; raising public awareness about dementia in LMICs would most likely take the form of our correct answer. Let’s go through our four options and find the community-based program that would best raise public awareness.

  1. Inform people that dementia is an abnormal condition rather than a normal part of aging. Right away, this comes across as a strong answer. We’re looking for the community-based program that would best raise public awareness. Informing people about dementia so they can recognize any signs or differences from normal aging is a great way to raise public awareness. 
  2. Educate people about all the disorders that can lead to dementia if they are left untreated. While educating people about the disorders helps inform the public, it’s not raising awareness as much as option A. Giving the public details about different disorders can certainly be helpful, but we’re focused on raising awareness to answer this question. Answer choice A does a better job of doing that. 
  3. Help people to distinguish between fluid intelligence and crystallized intelligence. This is similar to answer choice B where this knowledge is helpful, but it’s not necessarily raising awareness about dementia. The best way to raise awareness is to inform people about the disease and what they should look out for. A decline in fluid intelligence (reasoning and flexible thinking) is expected as you age, so explaining normal aging symptoms in this case would not be the best way to raise awareness.
  4. Explain to people that memory interference occurs more often than memory decay. This answer choice is out of scope. Once again, this can be helpful information so people understand how memories may be lost. However, within the context of this passage and this question, distinguishing between memory interference and memory decay is not going to raise awareness like informing people that dementia is an abnormal condition rather than a normal part of aging. Answer choice A is going to be our best answer. 

49) This is a passage-based question that, once again, focuses on one of the three disorders discussed in the passage. Previous question we focused on dementia, now we’re going into depression. We can focus on Paragraph 2 where the author talks about depression. The author says, “Studies suggest that structural factors, such as poverty or racial/ethnic minority status, are significant determinants of depression in LMICs.” We’re going to go through each of our four answer choices and define them-we ultimately want to find the one most consistent with dealing with structural factors like poverty or racial/ethnic minority status. 

  1. the life course perspective. Life-course theory analyzes people’s lives in the context of social, structural, psychological, and biological perspectives. The life-course approach to health is therefore going to look at environmental factors, life experiences, and social environment and how it affects health. That means looking into childhood, adolescence, adulthood, and all ages to tie it to health. The passage does not go into the different stages of life or how they tie into poverty and racial/ethnic minority status.
  2. social epidemiology. Social epidemiology considers social factors that were explicitly mentioned in the passage. Factors like race, ethnicity, or socioeconomic status. It studies the social determinants of health, which is exactly what we’re looking for. This is going to be our best answer choice so far.
  3. the biomedical model. A biomedical approach focuses on the biological or physical aspects of the illness. The biopsychosocial approach is the approach to healthcare that highlights the importance of the psychological and sociological study. The latter would be a better answer choice here. We can stick with answer choice B as our best answer.
  4. microsociology. Microsociology is the study of sociological day-to-day and face-to-face interactions of small-scale groups such as families, classes or individuals. The structural factors discussed in the passage deal with macrosociology. We can stick with answer choice C as our best answer choice.

50) This is the less-commonly seen setup to a question in the behavioral section of the exam. Often, the test-maker will reference something in the passage and we are asked to relate it to a vocabulary word: some psychology or sociology topic. In this case, we’re doing the opposite. We’re given a vocabulary word (medicalization) and we’re asked which passage statement it is most relevant to. 

Medicalization is the effort to describe a type of behavior as a symptom of an underlying illness that should be treated by a doctor. For example, there is an increasing effort to medicalize addiction and treat it as a disease of the brain. Medicalization has also been criticized as an attempt on the part of the powerful to control behaviors that are inconsistent with societal demands. Ultimately, we can say medicalization involves considering behaviors that weren’t previously medical conditions, as medical conditions.

  1. Research conducted with populations in LMICs is generally less extensive than research with populations of HICs. Medicalization does not have to do with research methods, but rather medicalization involves considering behaviors that weren’t previously medical conditions, as medical conditions. 
  2. Although commonly prescribed in HICs, antidepressant medications may not be supported by sufficient evidence to justify the cost in LMICs. While this is helpful information, this does not have to do with medicalization. Medicalization is the effort to describe a type of behavior as a symptom of an underlying illness that should be treated by a doctor. It does not deal with cost/benefit analysis.
  3. The prevalence of dementia is higher in HICs than in LMICs, which is likely because of underreporting in LMICs. Once again, this is helpful information, but underreporting does not always mean the disorder has not been medicalized. This is a superior answer to answer choices A and B which were not as relevant to our definition of medicalization. 
  4. When interventions from HICs are applied to LMICs, emphasis should be placed on framing excessive alcohol use as a treatable disorder. This is exactly what we’re looking for in an answer choice. Medicalization involves considering behaviors that weren’t previously medical conditions, as medical conditions. That’s what they’re trying to do with excessive alcohol use. By framing excessive alcohol use as a medical condition, it’s also framed as something you can treat. This is going to be our best answer choice. 

51) This is a passage-based question that, once again, focuses on one of the three disorders discussed in the passage. Previously we had a question that focused on dementia and a question that focused on depression. Now we’re jumping into a question that’s focusing on Paragraph 4 and excessive alcohol use. We’re jumping into the very last part of the passage where the author says, “excessive alcohol use is also associated with preventable injuries, which are more likely to result from binge drinking than alcohol dependence.” This is attributing preventable injuries to an effect of alcohol use. We can define each of our four answer choices and decide which is the correct effect.

  1. Tolerance. Alcohol tolerance involves the need to consume more alcohol to get the same desired effect that one was able to get previously with a lesser amount. This does not have to do with preventable injuries or the rowdy and risky behavior associated with binge drinking.
  2. Disinhibition. Disinhibition involves the inability to suppress unwanted or inappropriate behavior like the rowdy behavior and risk-taking associated with binge drinking. This is exactly what we’re looking for in our correct effect of alcohol use. This answer choice is superior to answer choice A. 
  3. Negative reinforcement. Negative reinforcers remove an aversive or unpleasant stimulus to increase or maintain the frequency of a behavior. This is out of scope as negative reinforcement is not mentioned in Paragraph 4 or when talking about preventable injuries that are more likely to result from binge drinking. 
  4. Positive reinforcement. Positive reinforcers add a wanted or pleasant stimulus to increase or maintain the frequency of a behavior. This is also out of scope as positive reinforcement is not mentioned in Paragraph 4 or when talking about preventable injuries that are more likely to result from binge drinking. We can stick with answer choice B as our best answer choice here.

52) The best way to answer behavioral questions that are set up this way is to go back to the passage and find the relevant information we need to answer the question. We will ultimately use our general knowledge to define the four terms given as answer choices and find the one that is most likely experienced by the caregivers. At the end of Paragraph 3, the author says, “Support for family caregivers of dementia patients is also essential in order to help caregivers balance the demands of caregiving with their other social responsibilities.” We want an answer choice that is consistent with the need to balance the demands of caregiving and other social responsibilities. 

  1. role strain. Role strain is the stress or strain experienced by an individual when incompatible behavior, expectations, or obligations are associated with a single social role. The author explicitly tells us the caregivers have to balance caregiving with other social responsibilities. The caregivers have more than one social role in this situation.
  2. relative deprivation. Relative deprivation refers to the feeling of disadvantage that arises when individuals compare themselves to others of similar status and feel that they possess relatively fewer resources and privileges. This is out of scope when considering this question and the caregivers of dementia patients. This question focuses on caregivers balancing the demands of caregiving with other social responsibilities.
  3. role conflict. Role conflict is a conflict between or among the roles corresponding to two or more statuses in one individual. This is exactly what is happening with the caregivers. The author says support is essential in order to help caregivers balance the demands of caregiving with their other social responsibilities. The caregivers experience role conflict and have to find a way to balance these multiple roles.
  4. relative poverty. Relative poverty is a measure of wealth inequality, describing an individual or group’s wealth relative to another individual or group. This is out of scope when considering this question and the caregivers of dementia patients. We can stick with answer choice C as our best option. 

Exam 4 P/S Solutions: Passage 10

53) This question is going to involve a hybrid of information we’ll get from the passage, and general content information. From the passage, we can focus on Paragraph 3 where the author talks about social capital being operationalized by dividing it into three component measures. One of these measures is collective efficacy. The author says, “Collective efficacy assessed respondents’ perception that members of their community were likely to help one another out.” We’ll keep this description in mind and also define self-efficacy which is a topic on AAMC’s content outline. Self-efficacy is a belief in one’s own competence and effectiveness. It’s how capable we believe we are of doing things and reaching specific goals. We’re going to pick an answer choice that’s consistent with both of these descriptions, but also consistent with the passage.

  1. Self-efficacy increases the risk of group polarization, whereas collective efficacy increases the risk of groupthink. Self-efficacy has to do with a belief in one’s own competence and effectiveness, while collective efficacy has to do with members of the community helping one another out. Group polarization is the tendency for groups to make decisions that are more extreme than the initial inclination of its members which is not related to self-efficacy. Groupthink is a process of reasoning or decision making by a group, especially one characterized by uncritical acceptance or conformity to a perceived majority view. The passage did not go over group decision-making.
  2. Self-efficacy is limited by the availability heuristic, whereas collective efficacy is limited by the bystander effect. Self-efficacy has to do with a belief in one’s own competence and effectiveness, while collective efficacy has to do with members of the community helping one another out. The availability heuristic is a faulty heuristic in which you decide based on information readily available to you, while the bystander effect is when someone is less likely to help another if other potential helpers are present. Neither of these is relevant to distinguishing self-efficacy from collective efficacy as described in the passage.
  3. Self-efficacy predicts an individual’s coping ability, whereas collective efficacy predicts a group’s prosocial behavior. Self-efficacy has to do with a belief in one’s own competence and effectiveness. This can predict an individual’s coping ability because believing in our own abilities can improve performance and how we attack tasks and work toward goals. Collective efficacy has to do with members of the community helping one another out. Prosocial behavior is voluntary behavior with the intent to help other people. These two descriptions are consistent with one another. Answer choice C is going to be our best answer so far.
  4. Self-efficacy prevents deindividuation in a group, whereas collective efficacy prevents peer pressure among individuals. Self-efficacy has to do with a belief in one’s own competence and effectiveness, while deindividuation happens when a person lets go of self-consciousness and control and does what the group is doing, usually with negative goals or outcomes. Collective efficacy has to do with members of the community helping one another out, while peer pressure is often used to describe instances where an individual feels indirectly pressured into changing their behavior to match that of their peers. Reasoning here is going to be similar to the reasoning we used in answer choice B. Neither of these concepts is relevant to distinguishing self-efficacy from collective efficacy, as described in the passage. Answer choice C is going to be our best answer choice.

54) This is a passage-based question that relies on specific details from Table 1. We can pull up Table 1 and the description of Table 1 here

We’re asked which variable is associated with a lower likelihood of having a mammogram, and the description tells us likelihood is expressed as odds ratios. There is increased likelihood for values over 1.00 and decreased likelihood for values below 1.00. We’re also told right below the table that an asterisk next to these odds indicates a statistically significant result. We can go through each of the variables listed as answer choices and answer according to what we see in Table 1. 

  1. Health insurance. While the odds ratio is statistically significant, it is above 1.00 meaning there is increased likelihood of mammography.
  2. Below poverty level. The odds ratio here is statistically significant (asterisk is present) and is below 1.00 meaning there is a decreased likelihood of having had a mammogram. This is going to be our correct answer choice. 
  3. Social cohesion. The odds ratio for social cohesion is not statistically significant and it is exactly 1.00 meaning there is an equal likelihood of mammography for this variable. 
  4. Social participation. The odds ratio for social participation is not statistically significant and it is above 1.00 meaning there is increased likelihood of mammography. We can stick with answer choice B as our best option.

55) This is a passage-based question that relies on specific details from Table 1. We’re going to attack this question similarly to how attacked the previous question in the set. We can pull up Table 1 and the description of Table 1 here:

We’re asked which component of social capital is most likely to affect mammography usage. The paragraph right above Table 1 tells us likelihood is expressed as odds ratios. There is increased likelihood for values over 1.00 and decreased likelihood for values below 1.00. We’re also told right below the table that an asterisk next to these odds indicates a statistically significant result. We’re essentially looking at the three options provided and finding the one that is furthest away from 1.00. The odds ratio that is furthest from 1.00 means the variable is most likely to affect mammography usage. 

  1. Social cohesion. This answer choice is exactly 1.00 and is not statistically significant so this is not a great option.
  2. Collective efficacy. Collective efficacy had an odds ratio of 1.40 and this is a statistically significant result. This is a better option than Social cohesion.
  3. Social participation. Social participation had an offs ratio of 1.05 and is not statistically significant so we cannot say it is likely to affect mammography usage. Option II, Collective efficacy, is the most likely to affect mammography usage.

Answer choice B correctly lists Option II as the component of social capital that is most likely to affect mammography usage.

56) This is a passage-based question and the author begins by telling us a researcher proposes an alternative to the hypothesis in the passage. In the passage, researchers were interested in the association between social capital and the odds of having had a mammogram in the past year because they hypothesized that mammography rates are partly due to social capital. This alternative hypothesis draws on the concept of institutional discrimination. Institutional discrimination targets specific, easily stereotyped, and generalizable attributes of individuals, such as race and gender. It often exists within governments, though it can also occur in any other type of social institution. Some countries around the world practice some form of institutionalized discrimination. For example, in some countries, women cannot vote, drive, or work certain jobs. We want to identify a prediction that is most likely in an alternative hypothesis that draws on this institutional discrimination. 

  1. Patients whose healthcare provider is from the same racial/ethnic group will have above average rates of mammography. This is describing racial concordance where there’s a shared identity between providers and patients. This does not draw on the concept of institutional discrimination which we described in our breakdown of the question.
  2. Mammography rates will be affected by healthcare providers who interact with patients differently according to racial/ethnic group. While this is drawing on the concept of discrimination, we’re looking for a hypothesis that draws on the concept of institutional discrimination, not individual discrimination.
  3. Metropolitan areas that are highly segregated across different racial/ethnic groups will have below average rates of mammography. Institutional discrimination targets generalizable attributes of individuals like race and gender, not the areas where people live. That better describes an alternative hypothesis that draws on the concept of segregation.
  4. Mammography rates will be affected by healthcare policies that have a disproportionate impact on racial/ethnic minority groups. This answer choice matches our prediction. We mentioned institutional discrimination targets specific, easily stereotyped, and generalizable attributes of individuals, such as race. Healthcare policies that have a disproportionate impact on racial/ethnic minority groups are consistent with institutional discrimination. Certain groups will be impacted disproportionately. This is going to be our best answer choice. 

Exam 4 P/S Solutions: Questions 57-59

57) This is a standalone question that’s similar to a lot of the questions in the behavioral section of the exam. We’re going to rely on knowing vocabulary to get to the right answer. We’re focused on immigrant health in this question. After moving from one culture to another, it takes time to learn a new set of cultural norms and acceptable behaviors. The process to acclimate to a new culture is called assimilation. Assimilation can occur in a variety of ways, including language acquisition and learning about the social roles and rules of the newly adopted culture. Eventually, the individual or immigrant group becomes a part of the new culture. The researcher is applying assimilation to immigrant health. We want to pick the answer choice that represents what the researcher expects about an immigrant group’s life expectancy. 

  1. surpass the majority group’s life expectancy over time. Assimilation involves the immigrant group acclimating to the new culture and eventually becoming a part of that new culture. We expect life expectancy to be in line with the majority group’s life expectancy over time. 
  2. approximate the majority group’s life expectancy over time. This answer choice is consistent with our breakdown. We expect life expectancy to be in line with the majority group’s life expectancy over time.
  3. fall behind the majority group’s life expectancy over time. Reasoning here is going to be the same as answer choice A. Assimilation involves the immigrant group acclimating to the new culture and eventually becoming a part of that new culture. We expect life expectancy to be in line with the majority group’s life expectancy over time. 
  4. maintain the same disparity with the majority group’s life expectancy over time. Initially there may be a disparity, but over time we expect life expectancy to be in line with the majority group’s life expectancy. We can stick with answer choice B as our best answer choice. 

58) This is a standalone question that’s similar to a lot of the questions in the behavioral section of the exam. We’re going to rely on knowing vocabulary to get to the right answer. We’re focused on symbolic interactionism.

Symbolic Interactionism focuses on small scale perspectives with small interactions between individuals. Through our interactions, we assign different meaning to objects and events, and these are different perspectives for different people. Social interactionism explains how individuals act in society, and can be expanded to look at the interactions of larger social groups to explain social change.

  1. social relationships. Individuals develop a sense of identity through social relationships. Through our social relationships and interactions, we assign different meaning to objects and events. This is a correct answer choice based on our breakdown of symbolic interactionism.
  2. reflective contemplation. This is an internal process and remember, symbolic Interactionism focuses on small scale perspectives with small interactions between individuals. There are no interactions with others in reflective contemplation. Answer choice A remains superior.
  3. cultural assimilation. Assimilation is the process by which an individual or group becomes part of a new culture and learns a new set of cultural norms and acceptable behaviors. This does not have to do with developing a sense of identity through interactions like we described in our breakdown of the question. 
  4. group therapy. Group therapy involves multiple people working with a therapist in a group. While helpful, this is not how individuals develop a sense of identity according to symbolic interactionism. Individuals develop a sense of identity through their interactions and social relationships. Answer choice A is our best answer. 

59) The organ of Corti is a structure in the cochlea of the inner ear which produces nerve impulses in response to sound vibrations. The organ of Corti is composed of hair cells held in place above the basilar membrane with their exposed short, hair-like stereocilia contacting or embedded in the tectorial membrane above them. All of the stereocilia are mechanoreceptors, and when bent by vibrations they respond by opening a gated ion channel. Said differently, mechanoreceptors in hair cells of the inner ear detect vibrations conducted from the eardrum.

  1. Mechanoreceptors detect mechanical forces. Mechanoreceptors in hair cells of the inner ear detect vibrations conducted from the eardrum. This is going to be our best answer choice. We can go through and briefly define our other answers to be thorough.
  2. Proprioceptors are sensory receptors that function in proprioception. Proprioception is the sense of the position of parts of the body, relative to other neighboring parts of the body. Mechanoreceptors remains our best answer choice. 
  3. Chemoreceptors detect the presence of chemicals. Mechanoreceptors remains our best answer choice.
  4. Osmoreceptors respond to the osmolarity of fluids. We can stick with answer choice A as our best option.